A parabola, with its vertex at (0,0), has a focus on the negative part of the y-axis. which statements about the parabola are true? select two options.

Answers

Answer 1

The true statement of the parabola is (a) directrix will cross through the positive part of the y-axis.

What are parabolas?

Parabolas are examples of a conic section such that it is formed by the intersection of a cone with a plane parallel to its side.

How to determine the true statements?

From the question, the given parameters are:

Vertex = (0,0)

Focus = Negative side of the y-axis

A parabola has quite a number of forms, one of these forms is the general form.

The general form is represented as:

(x - h)^2 = 4p(y - k)

The vertex of the parabola is

(h, k) = (0, 0).

So, we have:

(x - 0)^2 = 4p(y - 0)

Evaluate the difference

x^2 = 4py

Since the focus is on the negative side, the value of p will be negative.

Also, because the vertex is at the origin, the directrix of the parabola will cross through the positive part of the y-axis.

This means that the true statement of the parabola is (a) directrix will cross through the positive part of the y-axis.

Read more about parabola at:

https://brainly.com/question/4061870

#SPJ1

Complete question

A parabola, with its vertex at (0,0), has a focus on the negative part of the y-axis. Which statements about the parabola are true?

Check all that apply.

The directrix will cross through the positive part of the y-axis.

The equation of the parabola will be in the form y2 = 4px where the value of p is negative.

The equation of the parabola will be in the form x2 = 4py where the value of p is positive.

The equation of the parabola could be y2 = 4x.

The equation of the parabola could be x2 = y.


Related Questions

PLEASE HELP ASAP
If the Magnitude of Vector vec(w) is 48 and the direction is 235 degrees find vec(w) in component form.

Answers

If the magnitude of vector w is 48 and the direction is 235 degrees, we can find the vector w in component form by using trigonometry.

Let's denote the horizontal component as wx and the vertical component as wy.

The horizontal component, wx, can be found using the cosine of the angle:

wx = Magnitude × cos(Direction)

Substituting the given values:

wx = 48 × cos(235 degrees)

The vertical component, wy, can be found using the sine of the angle:

wy = Magnitude × sin(Direction)

Substituting the given values:

wy = 48 × sin(235 degrees)

Now we can calculate the values using a calculator or software. Rounding to two decimal places, we have:

wx ≈ 48 × cos(235 degrees) ≈ -32.73

wy ≈ 48 × sin(235 degrees) ≈ -32.00

Therefore, the vector w in component form is approximately (wx, wy) ≈ (-32.73, -32.00).

[tex]\huge{\mathfrak{\colorbox{black}{\textcolor{lime}{I\:hope\:this\:helps\:!\:\:}}}}[/tex]

♥️ [tex]\large{\textcolor{red}{\underline{\mathcal{SUMIT\:\:ROY\:\:(:\:\:}}}}[/tex]

let f be an automorphism of d4 such that f1h2 d. find f1v2.

Answers

So f(1v2) is the product of a reflection and rotation, specifically s * r^i+2.

To find f(1v2), we first need to determine the image of the generators of D4 under f. Let's denote the four generators of D4 as r, r^2, r^3, and s, where r represents a rotation and s represents a reflection.

Since f is an automorphism, it must preserve the group structure of D4. This means that f must satisfy the following conditions:

f(r * r) = f(r) * f(r)

f(r * s) = f(r) * f(s)

f(s * s) = f(s) * f(s)

f(1) = 1

From the first condition, we can see that f(r) must also be a rotation. Since there are only three rotations in D4 (r, r^2, and r^3), we can write:

f(r) = r^i

for some integer i. Note that i cannot be 0, since f must be a bijection (i.e., one-to-one and onto), and setting i = 0 would make f(r) equal to the identity element, which is not one-to-one.

From the second condition, we have:

f(r * s) = f(r) * f(s)

This means that f must map the product of a rotation and a reflection to the product of a rotation and a reflection. We know that rs = s * r^3, so we can write:

f(rs) = f(s * r^3) = f(s) * f(r^3)

Since f(s) must be a reflection, and f(r^3) must be a rotation, we can write:

f(s) = sr^j

f(r^3) = r^k

for some integers j and k.

Finally, from the fourth condition, we have:

f(1) = 1

This means that f must fix the identity element, which is 1.

Now, let's use these conditions to determine f(1v2):

f(1v2) = f(s * r) = f(s) * f(r) = (sr^j) * (r^i)

We know that sr^j must be a reflection, and r^i must be a rotation. The only reflection in D4 that can be expressed as the product of a reflection and a rotation is s * r^2, so we must have:

sr^j = s * r^2

j = 2

Therefore, we have:

f(1v2) = (sr^2) * (r^i) = s * r^2 * r^i = s * r^i+2

To know more about reflection and rotation,

https://brainly.com/question/15577335

#SPJ11

3x + 8y = -20
-5x + y = 19
PLS HELP ASAP

Answers

The system of equations are solved and x = -4 and y = -1

Given data ,

Let the system of equations be represented as A and B

where 3x + 8y = -20   be equation (1)

And , -5x + y = 19   be equation (2)

Multiply equation (2) by 8 , we get

-40x + 8y = 152   be equation (3)

Subtracting equation (1) from equation (3) , we get

-40x - 3x = 152 - ( -20 )

-43x = 172

Divide by -43 on both sides , we get

x = -4

Substituting the value of x in equation (2) , we get

-5 ( -4 ) + y = 19

20 + y = 19

Subtracting 20 on both sides , we get

y = -1

Hence , the equation is solved and x = -4 and y = -1

To learn more about equations click :

https://brainly.com/question/19297665

#SPJ1

Find the local maximum and minimum values and saddle point(s) of the function.
f(x, y) = x3 + y3 − 3x2 − 9y2 − 9x

Answers

The function f(x, y) = x³ + y³ - 3x² - 9y² - 9x has local maximum values at (-3, 0) and (1, 0), and a saddle point at (0, 3).

To find the critical points, we need to find the values of x and y where the partial derivatives of f with respect to x and y are equal to zero. Taking the partial derivatives, we get:

∂f/∂x = 3x² - 6x - 9 = 0

∂f/∂y = 3y² - 18y = 0

Solving these equations, we find the critical points to be (x, y) = (-3, 0), (1, 0), and (0, 3).

To determine the nature of these critical points, we can use the second partial derivative test. Computing the second partial derivatives:

∂²f/∂x² = 6x - 6

∂²f/∂y² = 6y - 18

∂²f/∂x∂y = 0

Substituting the critical points into the second partial derivatives, we find that:

∂²f/∂x²(-3, 0) = -24

∂²f/∂x²(1, 0) = -6

∂²f/∂x²(0, 3) = 0

Based on the sign of the second partial derivatives, we can determine the nature of each critical point. The point (-3, 0) has a negative second derivative, indicating a local maximum. The point (1, 0) has a negative second derivative, indicating a local maximum as well. Finally, the point (0, 3) has a second derivative equal to zero, indicating a saddle point.

Therefore, the function has local maximum values at (-3, 0) and (1, 0), and a saddle point at (0, 3).

Learn more about partial derivatives here:

https://brainly.com/question/6732578

#SPJ11

You select a marble without looking and then put it back. If you do this 9 times, what is the best prediction possible for the number of times you will pick a green or a pink marble?

Answers

The best prediction for the number of times you will pick a green or pink marble out of 9 selections is 2/9.

What is the best prediction for picking green or pink marble out of 9 selections?

To find the best prediction, we can assume that the marbles are equally likely to be selected each time.

Since there are two outcomes (green or pink) for each selection, the best prediction for the number of times you will pick a green or pink marble would be:

= 2 / 9

= 4.5.

Read more about Probability

brainly.com/question/24756209

#SPJ1

REALLY URGENT⚠️⚠️

FIND THE

Mean:

Median:

Mode:

Range:

in the 3 line plots!

Answers

Answer:mean for the first line is Mean x¯¯¯ 72

Median x˜ 73.5

Mode 48, 92

Range 44

Minimum 48

Maximum 92

Count n 12

Sum 864

Quartiles Quartiles:

Q1 --> 55

Q2 --> 73.5

Q3 --> 88.5

Interquartile

Range IQR 33.5

Outliers none

Step-by-step explanation:

find the standard form of the equation of the hyperbola with the given characteristics. vertices: (2, ±4) foci: (2, ±5)

Answers

The standard form of the equation of the hyperbola with the given characteristics is (x - 2)² / 16 - y² / 9 = 1

To find the standard form of the equation of a hyperbola, we need the coordinates of the center and either the distance between the center and the vertices (a) or the distance between the center and the foci (c).

Given the information:

Vertices: (2, ±4)

Foci: (2, ±5)

We can see that the center of the hyperbola is at (2, 0), which is the midpoint between the vertices. The distance between the center and the vertices is 4.

Since the foci are vertically aligned with the center, the distance between the center and the foci is 5.

The standard form of the equation of a hyperbola centered at (h, k) is:

(x - h)² / a² - (y - k)² / b² = 1

Since the foci and vertices are vertically aligned, the equation becomes:

(x - 2)² / a² - (y - 0)² / b² = 1

The value of a is the distance between the center and the vertices, which is 4, so a² = 4² = 16.

The value of c is the distance between the center and the foci, which is 5.

We can use the relationship between a, b, and c in a hyperbola:

c² = a² + b²

Solving for b²:

b² = c² - a² = 5² - 4² = 25 - 16 = 9

Therefore, b² = 9.

Substituting these values into the equation, we get:

(x - 2)² / 16 - y² / 9 = 1

So, the standard form of the equation of the hyperbola with the given characteristics is:

(x - 2)² / 16 - y² / 9 = 1

To know more about hyperbola refer here:

brainly.com/question/28989785#

#SPJ11

given that a and b are 4 × 4 matrices, deta=2, and det(2a−2bt )=1, find detb a 1/8 b 1/4 c 1/2 d 2 e 4

Answers

The value of det(b) cannot be determined based on the given information.

How to determine the value of det(b)?

To find det(b) based on the given information, let's analyze the equation det(2a - 2bt) = 1.

We know that det(2a - 2bt) = (2[tex]^n[/tex]) * det(a - bt), where n is the size of the matrix (in this case, n = 4).

Given that det(a) = 2, we can rewrite the equation as follows:

(2[tex]^n[/tex]) * det(a - bt) = 1

Substituting n = 4 and det(a) = 2, we have:

(2[tex]^4[/tex]) * det(a - bt) = 1

16 * det(a - bt) = 1

Now, we are given that det(a - bt) = 1, so we can rewrite the equation as:

16 * 1 = 1

This equation is not possible, as it contradicts the given information.

Therefore, there is no specific value that can be determined for det(b) based on the provided information.

Learn more about information

brainly.com/question/30350623

#SPJ11

surface area of triangular prism 5 in 4 in 8 in 2 in

Answers

The Total surface of triangular prism  is 112 inches.

Surface area calculation.

To calculate the surface area of a triangular prism, you need the measurements of the base and the height of the triangular faces, as well as the length of the prism.

The given measurements are;

Base ; 5 inches and  4 inches

height is 8 inches

Length of the prism is 2 inches.

To find the total surface area, we sum up the areas of all the faces:

Total surface area = area of triangular  faces + area of rectangular faces + area of lateral faces.

area of triangular faces = 5 inches × 4 inches = 20 inches.

area of the two faces = 20 ×2 =40

Area rectangular faces = 5 inches × 8 inches/ 2 = 40 inches.

Area of lateral faces = 8 inches ×2 = 16 square inches

for the two lateral faces is 16 × 2 = 32 square inches.

Total surface area = 40 square inches + 40 inches + 32 square inches = 112 square inches.

The Total surface of triangular prism  is 112 inches.

Learn more about surface area below.

https://brainly.com/question/16519513

#SPJ1

true/false. the equation y ′ 5xy = ey is linear.

Answers

False.  The equation is not linear because it contains a nonlinear term e^(y), which cannot be expressed as a linear combination of y and its derivatives.

A linear equation is one in which the dependent variable and its derivatives occur only to the first power and are not multiplied by any functions.

The given differential equation is y' = 5xy + ey. To determine whether it is a linear equation or not, we need to check if it satisfies the linearity property, i.e., whether it is a linear combination of y, y', and the independent variable x.

Here, we see that the term ey is not a linear combination of y, y', and x. Therefore, the given differential equation is not linear. If the term ey was absent, then the equation would be linear, and we could use standard methods to solve it, such as separation of variables or integrating factors. However, since ey is present, we cannot use these methods, and we need to use other techniques, such as power series or numerical methods.

In summary, the given differential equation y' = 5xy + ey is not linear since it contains a non-linear term ey.

Learn more about equation here:

https://brainly.com/question/29657983

#SPJ11

Consider the reduction of the rectangle. A large rectangle has a length of 16. 8 feet and width of 2. 3 feet. A smaller rectangle has a length of 4. 5 feet and width of x feet. Not drawn to scale Rounded to the nearest tenth, what is the value of x? 0. 1 feet 0. 6 feet 1. 6 feet 2. 0 feet.

Answers

A large rectangle has a length of 16.8 feet and width of 2.3 feet. A smaller rectangle has a length of 4.5 feet and width of x feet. the value of x is 0.6 feet

The solution of the given problem is as follows:

Given: A large rectangle has a length of 16.8 feet and width of 2.3 feet. A smaller rectangle has a length of 4.5 feet and width of x feet.

We know that the ratio of width is the same as the ratio of length of the rectangles of similar shape, thus the formula for the reduction of the rectangle is:

`large rectangle width / small rectangle width = large rectangle length / small rectangle length`

Putting the given values, we get:

`2.3 / x = 16.8 / 4.5`

Solving the above expression, we get:x = 0.6 feet (rounded to the nearest tenth)

Therefore, the value of x is 0.6 feet.Answer: 0.6 feet.

To know more about rectangle visit:

https://brainly.com/question/15019502

#SPJ11

Let A be a 8 times 9 matrix. What must a and b be if we define the linear transformation by T: R^a rightarrow R^b as T(x) = Ax ? a = ___________ b = __________

Answers

The required answer is a vector in R^5, then we would set b = 5.

To determine the values of a and b in the linear transformation defined by T(x) = Ax, we need to consider the dimensions of the matrix A and the vector x.

We know that A is an 8x9 matrix, which means it has 8 rows and 9 columns. We also know that x is a vector in R^a, which means it has a certain number of components or entries.
The matrix A has 8 rows and 9 columns, which means it maps 9-dimensional vector to 8-dimensional vectors .
To ensure that the matrix multiplication Ax is defined and results in a vector in R^b, we need the number of columns in A to be equal to the number of components in x. In other words, we need 9 = a and b will depend on the number of rows in A and the desired output dimension of T(x).

Therefore, a = 9 and b can be any number between 1 and 8, inclusive, depending on the desired output dimension of T(x). For example,

if we want T(x) to output a vector in R^5, then we would set b = 5.

To know more about linear transformation . Click on the link.

https://brainly.com/question/30514241

#SPJ11

Directions: Let f(x) = 2x^2 + x - 3 and g(x) = x - 1. Perform each function operation and then find the domain.

Problem: (f + g)(x)

Answers

Answer:

Domain is all real numbers

Step-by-step explanation:

First find function by adding

(2x^2+x-3)+(x-1)

2x^2+2x-4

For the situation below, identify the population and the sample and identify p and p if appropriate and what the value of p is. Would you trust a confidence interval for the true proportion based on these data? Explain briefly why or why not. The website of a certain newspaper asked visitors to the site to say whether they approved of recent bossnapping actions by workers who were outraged over being fired. Of those who responded, 54.9% said "Yes. Desperate times, desperate measures." What is the population? O A. All customers of the newspaper B. All visitors to the website C. All workers who were recently fired 0 D. All people on the internet Identify the sample. Choose the correct answer below. 0 A. The people on the internet who approved O B. The customers of the newspaper who responded ° C. The visitors to the website who approved O D. The visitors to the website who responded

Answers

The given options are:

A. All customers of the newspaper

B. All visitors to the website

C. All workers who were recently fired

D. All people on the internet

The population in this situation is the group of individuals that the study aims to generalize to. The population can be interpreted as the group of interest or the larger group to which the findings are intended to apply.

In this case, the population would most likely be option B: All visitors to the website. This is because the study is conducted on the website of a certain newspaper, and the responses are collected from the visitors to that specific website.

The sample, on the other hand, is the subset of individuals from the population that is actually surveyed or observed. It is used to gather information about the population.

The given options for the sample are:

A. The people on the internet who approved

B. The customers of the newspaper who responded

C. The visitors to the website who approved

D. The visitors to the website who responded

Based on the information provided, the sample would be option D: The visitors to the website who responded. These are the individuals who actively participated in the survey by providing their response on the website.

Regarding whether to trust a confidence interval for the true proportion based on these data, it would depend on the representativeness of the sample. If the sample is a random and representative sample of the population, then a confidence interval can provide a reasonable estimate of the true proportion. However, if there are concerns about the sampling method, sample size, or potential biases in the sample, it may not be advisable to fully trust the confidence interval.

Know more about population here;

https://brainly.com/question/27991860

#SPJ11

Si efectúan las operaciones indicadas ¿ cual es el valor de 1/2(1/2+3/2)?

Answers

Answer: 1

Step-by-step explanation:

0.5(0.5+1.5)=0.5*2=1

figure acfg below is a parallelogram if ag =2x+20 and cf =5x- 10, find the length of ag​

Answers

The solution is: the length of AG = 40.

Here, we have,

Lengths AG and CF of the parallelogram are equal.

i.e AG = CF

where AG = 2x + 20

         CF = 5x- 10

so, we get,

→ 2x + 20 = 5x-10

(collecting like terms): 5x - 2x = 20 + 10

→ 3x = 30

or, x=30÷3 = 10

∴ CF = 5x -10

        = 5(10) -10

        = 50 - 10

        = 40

and, AG = 2x + 20

              = 20 + 20

              = 40

∴ AG = 40 (answer)

Hence, The solution is: the length of AG = 40.

To learn more on parallelogram click:

brainly.com/question/6166074

#SPJ1

Let a,b,c be positive numbers. Find the volume of the ellipsoid
{ (x,y,z) ε R3 : x2/ a2 + y2/ b2 + z2/ c2 <1 } by fining a set Ω is subset of R3 whosevolume you know and an operator T ε τ (R3) such that T ( Ω ) equals theellipsoid above.

Answers

To find the volume of the ellipsoid { (x,y,z) ε R^3 : x^2/a^2 + y^2/b^2 + z^2/c^2 < 1 }, we can define a set Ω that has a known volume and an operator T that maps Ω to the ellipsoid.

Let's consider the set Ω to be the unit sphere centered at the origin, which has a volume of (4/3)π. Therefore, the volume of Ω is known.

Now, we can define the operator T as follows:

T : R^3 → R^3

T(x, y, z) = (ax, by, cz)

The operator T scales the coordinates of a point (x, y, z) by the factors a, b, and c, respectively.

To show that T(Ω) is equal to the ellipsoid, we need to prove two conditions:

T(Ω) is contained within the ellipsoid:

Let (x, y, z) be any point in Ω. Then, the squared norm of the transformed point T(x, y, z) is given by:

||T(x, y, z)||^2 = (ax)^2/a^2 + (by)^2/b^2 + (cz)^2/c^2 = x^2 + y^2 + z^2

Since x^2 + y^2 + z^2 < 1 for points in Ω, it follows that T(Ω) is contained within the ellipsoid.

The ellipsoid is contained within T(Ω):

Let (x, y, z) be any point in the ellipsoid, i.e., x^2/a^2 + y^2/b^2 + z^2/c^2 < 1.

We can scale the coordinates of this point by dividing them by a, b, and c, respectively, to obtain a point in Ω:

T^-1(x, y, z) = (x/a, y/b, z/c)

The squared norm of this transformed point is given by:

||T^-1(x, y, z)||^2 = (x/a)^2 + (y/b)^2 + (z/c)^2 = x^2/a^2 + y^2/b^2 + z^2/c^2 < 1

Therefore, the ellipsoid is contained within T(Ω).

Since both conditions are satisfied, we can conclude that T(Ω) is equal to the ellipsoid.

Finally, the volume of the ellipsoid can be determined by applying the operator T to the volume of Ω:

Volume of ellipsoid = Volume of T(Ω) = T(Volume of Ω)

= T((4/3)π)

= (4/3)π * a * b * c

Therefore, the volume of the ellipsoid is (4/3)π * a * b * c.

Learn more about ellipsoid here: brainly.com/question/32388486

#SPJ11

Determine the TAYLOR’S EXPANSION of the following function:
2
(1 + z)3 on the region |z| < 1.
Please show all work and circle diagrams.

Answers

The coefficients of the function (1 + z)^3 can be esxpressed as an infinite series:

(1 + z)^3 = 1 + 3z + 3z² + z³ + ...

The Taylor expansion of the function (1 + z)^3 on the region |z| < 1 can be obtained by applying the binomial theorem. The binomial theorem states that for any real number n and complex number z within the specified region, we can expand (1 + z)^n as a series of terms:

(1 + z)^n = C₀ + C₁z + C₂z² + C₃z³ + ...

To find the coefficients C₀, C₁, C₂, C₃, and so on, we use the formula for the binomial coefficients:

Cₖ = n! / (k!(n - k)!)

In this case, n = 3, and the region of interest is |z| < 1. To obtain the coefficients, we substitute the values of n and k into the binomial coefficient formula. After calculating the coefficients, we can express the function (1 + z)^3 as an infinite series:

(1 + z)^3 = 1 + 3z + 3z² + z³ + ...

By expanding the function using the binomial theorem and calculating the coefficients, we have obtained the Taylor expansion of (1 + z)^3 on the region |z| < 1.

Learn more about Taylor's expansion here:

https://brainly.com/question/32291388

#SPJ11

The perimeter of the scalene triangle is 54. 6 cm. A scalene triangle where all sides are different lengths. The base of the triangle, labeled 3 a, is three times that of the shortest side, a. The other side is labeled b. Which equation can be used to find the value of b if side a measures 8. 7 cm?.

Answers

The side b has a length of 19.8 cm.

To find the value of side b in the scalene triangle, we can follow these steps:

Step 1: Understand the information given.

The perimeter of the triangle is 54.6 cm.

The base of the triangle, labeled 3a, is three times the length of the shortest side, a.

Side a measures 8.7 cm.

Step 2: Set up the equation.

The equation to find the value of b is: b = 54.6 - (3a + a).

Step 3: Substitute the given values.

Substitute a = 8.7 cm into the equation: b = 54.6 - (3 * 8.7 + 8.7).

Step 4: Simplify and calculate.

Calculate 3 * 8.7 = 26.1.

Calculate (3 * 8.7 + 8.7) = 34.8.

Substitute this value into the equation: b = 54.6 - 34.8.

Calculate b: b = 19.8 cm.

By substituting a = 8.7 cm into the equation, we determined that side b has a length of 19.8 cm.

To know more about length, visit:

https://brainly.com/question/13118780

#SPJ11

Use the properties of addition and multiplication of real numbers given in Properties 2.3.1 to deduce that, for all real numbers a and b,
(i) a × 0 = 0 = 0 × a,
(ii) (-a)b = -ab = a(-b),
(iii) (-a)(-b) = ab.

Answers

We can prove that for all real numbers a and b (i) a × 0 = 0 = 0 × a, (ii) (-a)b = -ab = a(-b), and (iii) (-a)(-b) = ab.

Using the properties of addition and multiplication of real numbers given in Properties 2.3.1, we can prove the following

(i) For any real number a, we have

a × 0 = a × (0 + 0) (Property 2.3.1)

= a × 0 + a × 0 (Property 2.3.1)

Subtracting a × 0 from both sides, we get

a × 0 = 0 (Property 2.3.1)

Similarly, we can show that 0 × a = 0 using the same properties.

(ii) For any real numbers a and b, we have

(-a)b + ab = (-a + a)b (Property 2.3.1)

= 0 × b (Property 2.3.1)

= 0 (Part (i))

Subtracting ab from both sides, we get

(-a)b = -ab (Property 2.3.1)

Similarly, we can show that a(-b) = -ab using the same properties.

(iii) For any real numbers a and b, we have

(-a)(-b) + (-a)b = (-a)(-b + b) (Property 2.3.1)

= (-a) × 0 (Property 2.3.1)

= 0 (Part (i))

Subtracting (-a)b from both sides, we get

(-a)(-b) = ab (Property 2.3.1)

To know more about real numbers here

https://brainly.com/question/31715634

#SPJ4

help me please im stuck

Answers

The number of points Aiden earns for each visit is 2.5, so the total number of points he earns after v visits is:

Total points = 75 + 2.5v

In order to get a free movie ticket, he needs at least 90 points. Therefore, we can write the inequality:

75 + 2.5v ≥ 90

Simplifying and solving for v:

2.5v ≥ 15

v ≥ 6

Therefore, Aiden needs to make at least 6 visits to the movie theater to earn enough points for a free movie ticket. The inequality representing this is:

v ≥ 6

Prove that the function f : N × N → N defined as f(m, n) = 2^m 3^n is injective, but not surjective. (You are not allowed to use the factorization of integers into primes theorem, just use the properties that we know so far).

Answers

the function f : N × N → N defined as f(m, n) = 2^m 3^n is injective, but not surjective.

To prove that the function f : N × N → N defined as f(m, n) = 2^m 3^n is injective, we need to show that if f(m1, n1) = f(m2, n2), then (m1, n1) = (m2, n2). That is, if the function maps two distinct input pairs to the same output value, then the input pairs must be equal.

Suppose f(m1, n1) = f(m2, n2). Then, we have:

2^m1 3^n1 = 2^m2 3^n2

Dividing both sides by 2^m1, we get:

3^n1 = 2^(m2-m1) 3^n2

Since 3^n1 and 3^n2 are both powers of 3, it follows that 2^(m2-m1) must also be a power of 3. But this is only possible if m1 = m2 and n1 = n2, since otherwise 2^(m2-m1) is not an integer.

Therefore, the function f is injective.

To show that f is not surjective, we need to find an element in N that is not in the range of f. Consider the prime number 5. We claim that there is no pair (m, n) of non-negative integers such that f(m, n) = 5.

Suppose there exists such a pair (m, n). Then, we have:

2^m 3^n = 5

But this is impossible, since 5 is not divisible by 2 or 3. Therefore, 5 is not in the range of f, and hence f is not surjective.

To learn more about non-negative visit:

brainly.com/question/16155009

#SPJ11

find the limit (if it exists). (if an answer does not exist, enter dne.) lim t → 0 e4ti sin(2t) 2t j e−3tk

Answers

according to the question the limit is 2i + 1.

We can use L'Hopital's rule to evaluate this limit:

lim t → 0 e^4ti sin(2t) / (2t e^(-3t))

Taking the derivative of the numerator and denominator with respect to t, we get:

lim t → 0 [4i e^4ti sin(2t) + 2 e^4ti cos(2t)] / (2 e^(-3t) - 3t e^(-3t))

Plugging in t = 0, we get:

[4i + 2] / 2 = 2i + 1

what is L'Hopital's rule?

L'Hopital's rule is a mathematical theorem that provides a method to evaluate limits of indeterminate forms, which are expressions that cannot be directly evaluated by substitution. The rule states that if the limit of a quotient of two functions is an indeterminate form of type 0/0 or ∞/∞, then under certain conditions, the limit of the quotient of the derivatives of the numerator and denominator as x approaches the limit point is equal to the original limit.

To learn more about L'Hopital's rule visit:

brainly.com/question/29480665

#SPJ11

Nehemiah wrote that 4 + 4 = 8. Then he wrote that 4 + 4 – k = 8 – k. Select the phrases that make the statement true

Answers

To make the statement "4 + 4 = 8" true, the phrases that can be selected to make the subsequent statement "4 + 4 - k = 8 - k" true are "for any value of k" or "regardless of the value of k".

The initial statement "4 + 4 = 8" is true because the sum of 4 and 4 is indeed equal to 8.

In the subsequent statement "4 + 4 - k = 8 - k", we can see that both sides of the equation have subtracted the variable k. To make this statement true regardless of the value of k, we need to ensure that the subtraction of k on both sides does not affect the equality.

In other words, for any value of k, as long as we subtract the same value of k from both sides of the equation, the equation will remain true. Therefore, the phrases "for any value of k" or "regardless of the value of k" can be selected to make the statement "4 + 4 - k = 8 - k" true.

Learn more about statement here:

https://brainly.com/question/17238106

#SPJ11

Let A = [V1 V2 V3 V4 V5] be a 4 x 5 matrix. Assume that V3 = V1 + V2 and V4 = 2v1 – V2. What can you say about the rank and nullity of A? A. rank A ≤ 3 and nullity A ≥ 2 B. rank A ≥ 2 and nullity A ≤ 3 C. rank A ≥ 3 and nullity A ≤ 2 D. rank A ≤ 2 and nullity A ≥ 2 E. rank A ≥ 2 and nullity A ≤ 2

Answers

We have rank A ≤ 3 and nullity A ≥ 1. However, it is possible that the nullity is actually greater than 1 (for example, if V1 = V2 = V4 = 0), so the best answer is A. Rank A ≤ 3 and nullity A ≥ 2.

The rank of a matrix is the number of linearly independent rows or columns. From the given information, we can see that V3 is a linear combination of V1 and V2, and V4 is a linear combination of V1 and V2. This means that at least two of the rows (or columns) in A are linearly dependent, which implies that rank A ≤ 3.

The nullity of a matrix is the dimension of its null space, which is the set of all vectors that satisfy the equation Ax = 0 (where x is a column vector). Using the given information, we can rewrite the equation for V4 as 2V1 - V2 - V4 = 0, which means that any vector x that satisfies this equation (with the corresponding entries in x corresponding to V1, V2, and V4) is in the null space of A. This means that the nullity of A is at least 1.

Combining these results, we have rank A ≤ 3 and nullity A ≥ 1. However, it is possible that the nullity is actually greater than 1 (for example, if V1 = V2 = V4 = 0), so the best answer is A. rank A ≤ 3 and nullity A ≥ 2.

learn more about linear combination

https://brainly.com/question/30888143

#SPJ11

(1 point) find the absolute maximum and absolute minimum values of the function f(x)=x3−12x2−27x 9 over each of the indicated intervals.

Answers

To find the absolute maximum and minimum values of the function f(x) = x³ - 12x² - 27x + 9 over a given interval, we need to follow these steps:

1. Find the critical points of the function by setting its derivative f'(x) = 3x² - 24x - 27 equal to zero and solving for x. We get x = -3, 3, and 4 as critical points.

2. Evaluate the function at the critical points and the endpoints of the interval to find candidate points for the absolute max/min values.

f(-3) = -63, f(3) = -45, f(4) = 1, f(-infinity) = -infinity, and f(infinity) = infinity.

3. Compare the values of the function at the candidate points to determine the absolute maximum and minimum values.

The function has a local maximum at x = -3 and a local minimum at x = 4, but neither of these points is in the given interval. Therefore, we only need to consider the endpoints.

The absolute maximum value of the function over the interval (-infinity, infinity) is infinity, which occurs at x = infinity.

The absolute minimum value of the function over the interval (-infinity, infinity) is -infinity, which occurs at x = -infinity.

Explanation: We used the concept of critical points and candidate points to determine the absolute maximum and minimum values of the function over the given interval. The critical points are the points where the derivative of the function is zero or undefined, and the candidate points are the critical points and the endpoints of the interval. By evaluating the function at these points and comparing the values, we can identify the absolute max/min values. In this case, we found that the function has no absolute max/min values over the given interval, but has an absolute max of infinity at x = infinity and an absolute min of -infinity at x = -infinity over the entire domain of the function.    

To know more about function visit--

https://brainly.com/question/11624077

#SPJ11

Find the following for the given equation. r(t) = e−t, 2t2, 3 tan(t) (a) r'(t) = (b) r''(t) = (c) Find r'(t) · r''(t). 5. Find the following for the given equation. r(t) = 3 cos(t)i + 3 sin(t)j (a) r'(t) = (b) r''(t) = (c) Find r'(t) · r''(t).

Answers

(a) For the equation r(t) = e^(-t), 2t^2, 3tan(t), the first derivative is r'(t) = -e^(-t), 4t, 3sec^2(t). (b) The second derivative is r''(t) = e^(-t), 4, 6tan(t)sec^2(t). (c) The dot product of r'(t) and r''(t) is (-e^(-t))(e^(-t)) + (4t)(4) + (3sec^2(t))(6tan(t)sec^2(t)) = -e^(-2t) + 16t + 18tan(t)sec^4(t).

(a) For the equation r(t) = 3cos(t)i + 3sin(t)j, the first derivative is r'(t) = -3sin(t)i + 3cos(t)j.

(b) The second derivative is r''(t) = -3cos(t)i - 3sin(t)j.

(c) The dot product of r'(t) and r''(t) is (-3sin(t))(-3cos(t)) + (3cos(t))(3sin(t)) = 0, which means that the vectors r'(t) and r''(t) are orthogonal or perpendicular to each other.

Learn more about first derivative here

https://brainly.com/question/14619629

#SPJ11

Jean’s girl scout troop is selling cookies. The number of boxes of Thin Mints that they sold was 14 times the number of boxes of S’mores cookies they sold. If they sold 47 boxes of S’mores cookies, how many Thin Mints boxes did they sell?

Answers

658 boxes of thin mints 47x14=658

what is the total area between f(x)=−6x and the x-axis over the interval [−4,2]?

Answers

The total area between the function f(x) = -6x and the x-axis over the interval [-4, 2] is -60 square units.

To find the total area between the function f(x) = -6x and the x-axis over the interval [-4, 2], we need to calculate the definite integral of the absolute value of the function over that interval.

Since the function f(x) = -6x is negative for the given interval, taking the absolute value will yield the positive area between the function and the x-axis.

The integral to find the total area is:

∫[-4, 2] |f(x)| dx

Substituting the function f(x) = -6x:

∫[-4, 2] |-6x| dx

Breaking the integral into two parts due to the change in sign at x = 0:

∫[-4, 0] (-(-6x)) dx + ∫[0, 2] (-6x) dx

Simplifying the integral:

∫[-4, 0] 6x dx + ∫[0, 2] (-6x) dx

Integrating each part:

[tex][3x^2] from -4 to 0 + [-3x^2] from 0 to 2[/tex]

Plugging in the limits:

[tex](3(0)^2 - 3(-4)^2) + (-3(2)^2 - (-3(0)^2))[/tex]

Simplifying further:

[tex](0 - 3(-4)^2) + (-3(2)^2 - 0)[/tex]

(0 - 3(16)) + (-3(4) - 0)

(0 - 48) + (-12 - 0)

-48 - 12

-60

Therefore, the total area between the function f(x) = -6x and the x-axis over the interval [-4, 2] is -60 square units. Note that the negative sign indicates that the area is below the x-axis.

To know more about function refer to-

https://brainly.com/question/12431044

#SPJ11

quizletmeasures of central tendency include all except: a. standard deviation b. median c. mean d. mode

Answers

Answer:

a. standard deviation

Step-by-step explanation:

Standard deviation measures the variation (how spread out the data is from the mean) of a data set.  

Other Questions
Which of the following is not a characteristic of both experiments andsurveys?A. The researcher exercises direct control over at least one variable.B. Subjects should be randomly selected.C. Data are collected about a population.D. The data collected might be studied for patterns. Does hydrogen have similar physical properties to alkali metals? Ann wants to spend at most \$8.50 on school supplies. She needs to purchase a binder ( costs $6.89) and wants to spend the rest on pens, which cost $ 0.59 each. Which inequality represents this situation, where x is the number of pens Ann can buy?A. 0.59x + 6.89 < 8.50B. 0.59x + 8.50 < 6.89C. 6.89x + 0.59 < 8.50D. 8.50x + 6.89 < 0.59Please help who is the father of computer?1.Charles Babbage 2.James Waat Determine the missing card value that will result in a product of -324.-6 3 -9 Please I have only 13 HOURS TO FINISH MY SEMESTER9. you don't need to answer all of them just someWhy do storytellers and historians sometimes alter the facts when recounting a story?What is the difference between subjective and objective writing?Explain how the resolution in Julie of the Wolves helps develop one of the novel's themes.What skills do you need to participate successfully in a collaborative discussion about an assigned novel?What skills do you need to participate successfully in a collaborative discussion about an assigned novel? When a system fails it _____ our other systems causing us to be sick.A. connects B. repairsC. affects D. structures Mrs. Norton wants to buy health insurance for her family, Plan A costs $620.50 per month. Plan B costs $597.20 per month. How much will she save in 12 months by choosing Plan B instead ofPlan A?A $7,446.00 B $7,166.40C $320.60D $279.60 What is the expected value of each option? Does anybody know how to do this? ANSWER QUICKLY THIS IS A TIMED TEST The community garden in your neighborhood has threedifferent types of vegetable plants: peppers, tomatoes, andcucumbers. The ratio of the number of peppers to tomatoesto cucumbers is 1:5:3. If there are 48 cucumber plants in thegarden, how many total vegetable plants are there in thegarden? Which is the formula for nitrogen trihydride? NH3 N3H 3NH N3H3NH3N3H3NHN3H3 Simplify this expression.(2 - 5/6) 3/2O 6V2 - 157662 - 15/12O 672-876O 672-30/3 Which of the following statements is not true? Many Fortune 500 U.S. firms derive more than half their revenues from foreign operations. Technology companies are particularly dependent on offshore revenue. Eighty percent of the toys sold in the United States are manufactured in China. Most PCs manufactured in China use microprocessor chips manufactured in Korea. In 2017, 60 percent of Apple's revenue was generated outside of the United States. Find the length of the hypotenuse if a = 5 and b = 7. Round to the nearest tenths. Keith has saved 12 quarters from washing cars how many cents does Keith have What incident serves to initiate the conflict in Hamlet?(I need a summary) 18/20 + 12/20 (simpfiy it ) What is the volume of a cone 22yd by 22yds help please ill give extra points